Dermatology Multiple Choice Questions PDF

Loading...
Loading...
Loading...
Loading...
Loading...
Loading...
Loading...

Document Details

SelfDeterminationSerpentine

Uploaded by SelfDeterminationSerpentine

Tags

dermatology medical skin conditions biology

Summary

This document contains multiple choice questions on dermatology, covering various topics such as skin layers, cell types, conditions like psoriasis and pemphigus, and diagnostic techniques such as diascopy and Woods Lamp examination.

Full Transcript

Memrizz DERMATOLOGY Multiple Choice Questions What is the main characteristic of the Stratum Corneum? a) Contains keratinocyte rich tissue. b) Has active mitotic cells. c) Features desmosomes for attachment. d) Composed of dead flat cells. Correct answer: Compose...

Memrizz DERMATOLOGY Multiple Choice Questions What is the main characteristic of the Stratum Corneum? a) Contains keratinocyte rich tissue. b) Has active mitotic cells. c) Features desmosomes for attachment. d) Composed of dead flat cells. Correct answer: Composed of dead flat cells. Where is the Stratum Lucidum primarily located? a) On the scalp and forehead. b) In the upper chest area. c) On the back and abdomen. d) In the palms and soles. Correct answer: In the palms and soles. What is a key function of the Stratum Granulosum? a) Serves as a barrier against UV light. b) Produces Filaggrin and keratohyalin granules. c) Contains active mitotic cells. d) Is responsible for sweat production. Correct answer: Produces Filaggrin and keratohyalin granules. Which structure aids in the attachment of keratinocytes in the Stratum Spinosum? a) Langerhans cells. b) Melanocytes. c) Desmosomes. d) Sebaceous glands. Correct answer: Desmosomes. What is a defining trait of the Stratum Basale? a) High mitotic activity. b) Contains dead keratinized cells. c) Features refractile granules. d) Is located below the epidermis. Correct answer: High mitotic activity. What does Epidermal Turnover Time (ETT) measure? a) Rate of sweat production. b) Time for skin to regenerate. c) Migration duration of keratinocytes. d) Frequency of cell mitosis. Correct answer: Migration duration of keratinocytes. What is the average epidermal turnover time for healthy skin? a) 2 weeks (14 days). b) 1 month (30 days). c) 8 weeks (42 to 75 days). d) 6 weeks (36 days). Correct answer: 8 weeks (42 to 75 days). How does epidermal turnover change in conditions like Psoriasis? a) It takes 10 days. b) It lasts for 2 weeks. c) It is unchanged from normal. d) It occurs in 4 days. Correct answer: It occurs in 4 days. What type of cells are Langerhans cells? a) Melanin pigment producing cells b) Mechanoreceptors for touch c) Antigen presenting cells d) Ectodermal origin cells Correct answer: Antigen presenting cells Where are Melanocytes located? a) Stratum spinosum b) Stratum basale c) Stratum corneum d) Stratum granulosum Correct answer: Stratum basale From where do Langerhans cells originate? a) Neural crest b) Ectoderm c) Bone marrow d) Stratum basale Correct answer: Bone marrow Which granules are found in Langerhans cells? a) Melanosomes b) Neurosecretory granules c) Keratin filaments d) Birbeck granules Correct answer: Birbeck granules What is the primary function of Merkel cells? a) Mechanoreceptors for touch b) Antigen presenting c) Melanin production d) Barrier formation Correct answer: Mechanoreceptors for touch Which condition is associated with Langerhans cells? a) Langerhans cell histiocytosis b) Melanoma c) Pemphigus d) Merkel cell carcinoma Correct answer: Langerhans cell histiocytosis What is the most specific marker for Langerhans cells? a) S100 b) CD1a c) Cytokeratin-20 d) Q Correct answer: Q What do melanocytes produce? a) Birbeck granules b) Cytokeratin c) Melanin pigment d) Neurosecretory granules Correct answer: Melanin pigment Which marker is used to identify Merkel cells? a) S100 b) Melan-A c) Cytokeratin-20 d) HMB 45 Correct answer: Cytokeratin-20 What causes the defect associated with pemphigus? a) Melanin production failure b) Langerhans cells dysregulation c) Defect in Desmoglein (Dsg) d) Merkel cell dysfunction Correct answer: Defect in Desmoglein (Dsg) What cellular structure is essential for cell adhesion? a) Intra epidermal / Intercellular K-K bridges b) Basement membrane c) Desmosomes d) Neurosecretory vesicles Correct answer: Intra epidermal / Intercellular K-K bridges What appearance is associated with pemphigus? a) Granular appearance b) Row of tombstone appearance c) Stratified appearance d) Papillomatous appearance Correct answer: Row of tombstone appearance What is the Dermo-Epidermal Junction? a) Layer between the epidermis and subcutaneous tissue. b) Zone of fat deposition beneath the dermis. c) Area rich in blood vessels and nerves. d) Specialized zone connecting the epidermis and dermis. Correct answer: Specialized zone connecting the epidermis and dermis. How is the dermis divided? a) Into superficial and deep layers. b) Into external and internal layers. c) Into epidermis and hypodermis. d) Into papillary and reticular layers. Correct answer: Into papillary and reticular layers. What constitutes the majority of the dermis? a) Reticular Dermis. b) Papillary Dermis. c) Subcutaneous Layer. d) Epidermis. Correct answer: Reticular Dermis. What is another name for the subcutaneous layer? a) Hypodermis. b) Dermis. c) Adipose layer. d) Epidermis. Correct answer: Hypodermis. What is panniculitis? a) Inflammation of the subcutaneous layer. b) Infection of the epidermis. c) Swelling in the dermis. d) Bruising of the skin. Correct answer: Inflammation of the subcutaneous layer. What type of panniculitis is associated with erythema nodosum? a) Lobular Panniculitis. b) Septal Panniculitis. c) Dermatitis. d) Folliculitis. Correct answer: Septal Panniculitis. Which pathological condition involves hyperkeratosis? a) Inflammation of skin layers. b) Retained nuclei in corneum. c) Intercellular edema. d) Pathological thickening of the skin. Correct answer: Pathological thickening of the skin. What is ballooning degeneration characterized by? a) Inflammation in the dermis. b) Thickening of the stratum corneum. c) Loss of skin elasticity. d) Intracellular edema within the stratum spinosum. Correct answer: Intracellular edema within the stratum spinosum. Define parakeratosis. a) Excessive skin shedding. b) Infection of the upper skin. c) Retention of the nucleus in the stratum corneum. d) Degeneration of fat lobules. Correct answer: Retention of the nucleus in the stratum corneum. What is spongiosis? a) Thickening of the stratum corneum. b) Presence of intercellular edema between cells. c) Breakdown of keratinocytes. d) Accumulation of dead skin cells. Correct answer: Presence of intercellular edema between cells. What shape do acantholytic cells have? a) Oval-shaped cells. b) Cuboidal-shaped cells. c) Irregular-shaped cells. d) Columnar-shaped cells. Correct answer: Oval-shaped cells. What feature is associated with the nucleus of acantholytic cells? a) Large nucleus. b) Small nucleus. c) Absence of nucleus. d) Fragmented nucleus. Correct answer: Large nucleus. What does the perinuclear halo indicate? a) Absence of cytoplasm. b) Increased keratin production. c) Presence of a halo around the nucleus. d) Presence of bacteria. Correct answer: Presence of a halo around the nucleus. How can acantholytic cells be identified? a) Through a biopsy with a punch method. b) By examining blood samples under a microscope. c) Using a skin scraping technique. d) Using a Tzanck smear with Giemsa stain. Correct answer: Using a Tzanck smear with Giemsa stain. Which autoimmune disorder is associated with acantholytic cells? a) Psoriasis vulgaris. b) Pemphigus group. c) Atopic dermatitis. d) Lupus erythematosus. Correct answer: Pemphigus group. What is a common bacterial infection associated with acantholytic cells? a) Tinea corporis. b) Staphylococcal Scalded Skin Syndrome. c) Basal cell carcinoma. d) Atypical dermatitis. Correct answer: Staphylococcal Scalded Skin Syndrome. Which viral infection is linked to acantholytic cells? a) Human Papillomavirus (HPV) infection. b) Varicella-Zoster Virus (VZV) infection. c) Herpes Simplex Virus (HSV) infection. d) Cytomegalovirus (CMV) infection. Correct answer: Herpes Simplex Virus (HSV) infection. What do Blaschko's lines represent? a) Skin healing processes after injury. b) Lymphatic drainage patterns in the skin. c) Blood supply routes in the dermis. d) Epidermal cell migration pathways during development. Correct answer: Epidermal cell migration pathways during development. Why are Langer's lines important in dermatology? a) They indicate areas of high blood supply. b) They show the distribution of nerve endings. c) They help minimize scarring during surgeries. d) They define the limits of skin elasticity. Correct answer: They help minimize scarring during surgeries. What characterizes RSTL (Relaxed Skin Tension Lines)? a) They run parallel to the major blood vessels. b) Do not correspond to arteries, veins, or nerves. c) They indicate nerve innervation of the skin. d) They are purely horizontal or vertical lines. Correct answer: Do not correspond to arteries, veins, or nerves. In which area are RSTL characterized as being V-shaped? a) On the upper spine. b) On the forearms. c) On the lower back. d) On the forehead. Correct answer: On the upper spine. How should incisions be made during surgery to promote healing? a) Across the tension lines at random angles. b) Deeply within the dermal layer regardless of orientation. c) Along or parallel to skin tension lines. d) Only in horizontal lines for aesthetic reasons. Correct answer: Along or parallel to skin tension lines. What is the definition of diascopy? a) A technique known as the Vitro Pressure Test. b) A method for taking biopsies. c) A procedure for measuring skin elasticity. d) A type of allergy skin testing. Correct answer: A technique known as the Vitro Pressure Test. What does the diascopy procedure involve? a) Injecting a dye under the skin. b) Taking a photograph of the lesion. c) Scraping the top layer of the skin. d) Pressing the lesion with a glass slide. Correct answer: Pressing the lesion with a glass slide. What is a primary use of diascopy in dermatology? a) Diagnosing systemic diseases. b) Evaluating hair loss patterns. c) Assessment of skin lesions. d) Determining skin moisture levels. Correct answer: Assessment of skin lesions. What is erythema? a) A skin infection caused by bacteria. b) A type of fungal skin lesion. c) Redness of the skin indicating various conditions. d) A common allergic reaction to medications. Correct answer: Redness of the skin indicating various conditions. What indicates a non-blanching response in erythema? a) Redness fades under pressure. b) Skin turns pale when pressed. c) Redness persists, suggesting underlying severe condition. d) No redness observed at all. Correct answer: Redness persists, suggesting underlying severe condition. What is purpura? a) Purple spots on skin from RBC extravasation. b) A type of skin rash from allergens. c) Patches of dry skin with redness. d) Yellow-colored spots on the skin. Correct answer: Purple spots on skin from RBC extravasation. Apple jelly nodules are associated with which condition? a) Psoriasis. b) Contact dermatitis. c) Acne vulgaris. d) Lupus Vulgaris. Correct answer: Lupus Vulgaris. What is the primary purpose of a Woods Lamp Examination? a) To treat fungal infections on the skin. b) To measure skin hydration levels. c) To diagnose skin conditions through fluorescence. d) To remove skin lesions non-invasively. Correct answer: To diagnose skin conditions through fluorescence. What is the wavelength range for Woods Lamp Examination? a) 400-410 nm. b) 320-330 nm. c) 350-355 nm. d) 360-364 nm. Correct answer: 360-364 nm. Which filter composition is used in a Woods Lamp? a) 10% zinc oxide and aluminum oxide. b) 5% copper oxide and glass. c) 15% magnesium oxide and quartz. d) 9% nickel oxide and barium silicate. Correct answer: 9% nickel oxide and barium silicate. What color fluorescence does Microsporum exhibit under Woods Lamp? a) Blue-green fluorescence. b) Dull blue fluorescence. c) Yellow fluorescence. d) Coral red fluorescence. Correct answer: Blue-green fluorescence. What condition displays yellow fluorescence under Woods Lamp? a) Pityriasis Versicolor. b) Trichophyton Schonleinii. c) Corynebacterium Minutissimum. d) Vitiligo. Correct answer: Pityriasis Versicolor. Corynebacterium Minutissimum under Woods Lamp produces what color? a) Yellow fluorescence. b) Milky white appearance. c) Dull blue fluorescence. d) Coral red fluorescence. Correct answer: Coral red fluorescence. How does vitiligo appear under a Woods Lamp? a) As bright red lesions. b) As dull yellow spots. c) As milky white. d) As dark brown patches. Correct answer: As milky white. What is the procedure in Tzanck Smear? a) Directly injecting a dye into the lesion. b) Deroofing a fluid-filled lesion for scraping. c) Applying topical antibiotics to the affected area. d) Measuring skin temperature around the lesion. Correct answer: Deroofing a fluid-filled lesion for scraping. Which stain is used for microscopic examination in Tzanck Smear? a) Hematoxylin and eosin. b) Crystal violet stain. c) Giemsa stain. d) Silver nitrate solution. Correct answer: Giemsa stain. What are acantholytic cells associated with? a) Eczema and psoriasis. b) Skin cancer and basal cell carcinoma. c) Bacterial infections and fungal diseases. d) Pemphigus group and Herpes infections. Correct answer: Pemphigus group and Herpes infections. Multinucleate giant cells are related to which conditions? a) HSV types 1 and 2, and Varicella infections. b) Acne vulgaris and dermatitis. c) Shingles and ringworm. d) Seborrheic keratosis and molluscum contagiosum. Correct answer: HSV types 1 and 2, and Varicella infections. What agent is used in cryotherapy? a) Liquid nitrogen at -196 Celsius. b) Helium gas at -150 Celsius. c) Carbon monoxide at -100 Celsius. d) Chlorine gas at -50 Celsius. Correct answer: Liquid nitrogen at -196 Celsius. What is the mechanism of cryotherapy? a) Heats tissue to stimulate growth. b) Freezes tissue leading to cellular death. c) Applies radiation to enhance healing. d) Increases blood flow to the area. Correct answer: Freezes tissue leading to cellular death. Which conditions is cryotherapy indicated for? a) Melanoma and skin tags. b) Acne and rosacea. c) Warts and keloids. d) Sunburn and eczema. Correct answer: Warts and keloids. What wavelength range is used in Narrow Band UVB therapy? a) 400-450 nm. b) 250-300 nm. c) 311 ± 2 nm. d) 350-370 nm. Correct answer: 311 ± 2 nm. PUVA therapy combines psoralen with which type of therapy? a) UVA therapy. b) UVB therapy. c) Cryotherapy. d) Electroconvulsive therapy. Correct answer: UVA therapy. What is the indication for PUVA therapy? a) Psoriasis, vitiligo, and atopic dermatitis. b) Moles and warts. c) Bacterial infections. d) Skin cancer treatment. Correct answer: Psoriasis, vitiligo, and atopic dermatitis. What is a characteristic example of intraepidermal blistering disorders? a) Pemphigus group of disorders. b) Epidermolysis bullosa. c) Dermatitis herpetiformis. d) Bullous pemphigoid. Correct answer: Pemphigus group of disorders. Which disorders fall under subepidermal blistering disorders? a) Psoriasis and eczema. b) Acne and rosacea. c) Acantholytic disorders. d) Pemphigoid group and dermatitis herpetiformis. Correct answer: Pemphigoid group and dermatitis herpetiformis. Where does the pemphigus group of disorders split in the skin? a) Subepidermal. b) Dermal layer. c) Epidermal-dermal junction. d) Intraepidermal. Correct answer: Intraepidermal. Which proteins are targeted in pemphigus disorders? a) Collagen fibers. b) Keratin filaments. c) Elastic fibers. d) Desmosomes connecting keratinocytes. Correct answer: Desmosomes connecting keratinocytes. What is the basis for classification of immunobullous disorders? a) Type of bacteria involved. b) Size of the blisters formed. c) The level of the split in the skin. d) Age of the patient. Correct answer: The level of the split in the skin. What defines mechanobullous disorders? a) Caused by nutritional deficiencies. b) Related to mechanical factors. c) Genetic mitochondrial issues. d) Allergic reactions to medications. Correct answer: Related to mechanical factors. Inherited acantholytic disorders can be characterized by which factor? a) Genetic predisposition. b) Environmental causes. c) Viral infections. d) Autoimmune responses. Correct answer: Genetic predisposition. What is the definition of acantholysis? a) A type of skin ulceration. b) An autoimmune skin condition. c) The process of ruptured or separated desmosomes. d) Inflammation of the epidermis. Correct answer: The process of ruptured or separated desmosomes. What are desmosomes responsible for in the skin? a) Regulating blood flow in the epidermis. b) Producing skin collagen. c) Providing adhesion between keratinocytes. d) Protecting skin from UV rays. Correct answer: Providing adhesion between keratinocytes. Which antibody is primarily associated with pemphigus foliaceous? a) Desmoglein-3 antibodies. b) IgE antibodies. c) Desmoglein-1 antibodies. d) Autoantibodies against collagen. Correct answer: Desmoglein-1 antibodies. Where does pemphigus foliaceous mainly affect the skin? a) The lower epidermis. b) The dermal layer. c) The upper epidermis. d) The mucosal layer. Correct answer: The upper epidermis. What is a characteristic clinical feature of pemphigus vulgaris? a) Absence of skin split. b) Upper epidermis involvement only. c) Presence of oral lesions in 95% of cases. d) Only affects scalp and chest. Correct answer: Presence of oral lesions in 95% of cases. What is indicated by a positive Nikolsky sign? a) Formation of blisters on the skin. b) Separation of epidermal layers upon pressure. c) Presence of Tzanck cells under a microscope. d) A weak immune response to allergens. Correct answer: Separation of epidermal layers upon pressure. What does the Bulla Spread Sign indicate? a) The healing of active lesions. b) Extension of bulla margin upon lateral pressure. c) Absence of acantholytic cells. d) A positive reaction to allergens. Correct answer: Extension of bulla margin upon lateral pressure. Where are common sites for lesions in pemphigus? a) Solitary hair follicles only. b) Feet and palms only. c) Scalp, oral mucosa, and trunk. d) Exclusively on the face. Correct answer: Scalp, oral mucosa, and trunk. What is tested in a Tzanck smear? a) Presence of Tzanck cells in a vesicle. b) Bacterial cultures from lesions. c) Histopathology of the dermis. d) Levels of antibodies in blood. Correct answer: Presence of Tzanck cells in a vesicle. What characteristic is observed in Pemphigus Vulgaris histopathology? a) Suprabasal split resembling a row of tombstones. b) Thickened dermal layers. c) Minimal presence of acantholytic cells. d) Basal cell layer is fully intact. Correct answer: Suprabasal split resembling a row of tombstones. What is the mainstay treatment for Pemphigus Vulgaris? a) Topical antibiotic ointments. b) Systemic steroids (vigorous). c) Regular use of salicylic acid. d) Daily UV light exposure. Correct answer: Systemic steroids (vigorous). What additional treatment requires urine monitoring? a) Rituximab. b) Prednisolone. c) Azathioprine. d) Cyclophosphamide. Correct answer: Cyclophosphamide. What is the purpose of Rituximab in treating pemphigus? a) To target CD20 on B cells. b) To reduce inflammation in the skin. c) To heal oral lesions quickly. d) To stimulate collagen production. Correct answer: To target CD20 on B cells. What is the primary definition of acantholysis? a) The process by which desmosomes are ruptured. b) A type of skin infection. c) A condition causing skin pigmentation. d) Destruction of keratinocytes. Correct answer: The process by which desmosomes are ruptured. Which antibodies characterize Pemphigus Foliaceous? a) Desmoglein-3 antibodies. b) Desmoglein-1 antibodies. c) Collagen-4 antibodies. d) BPAG 2 antibodies. Correct answer: Desmoglein-1 antibodies. Where is Desmoglein-1 predominantly found? a) Lower epidermis only. b) Upper epidermis and oral mucosa. c) Subepidermal layers. d) Only in the hair follicles. Correct answer: Upper epidermis and oral mucosa. What type of split is associated with Desmoglein-3? a) Subcorneal split. b) Intradermal split. c) Suprabasal split. d) Epidermal split. Correct answer: Suprabasal split. What is the Nikolsky Sign? a) Formation of blisters upon scratch. b) Swelling of skin upon touch. c) Skin layers separate with tangential pressure. d) Erythema following pressure application. Correct answer: Skin layers separate with tangential pressure. What characterizes the Pemphigoid group of disorders? a) Defects in Basement Membrane Zone proteins. b) Destruction of keratinocyte adhesion. c) Accumulation of eosinophils. d) Inflammation of superficial layers. Correct answer: Defects in Basement Membrane Zone proteins. Which autoantibody is deposited in Pemphigoid? a) IgM. b) IgA. c) IgE. d) IgG. Correct answer: IgG. At what age range do Pemphigoid disorders primarily affect individuals? a) 20 to 40 years. b) 30 to 50 years. c) 50 to 70 years. d) 60 to 80 years. Correct answer: 60 to 80 years. How is the Nikolsky Sign observed in Pemphigus Vulgaris? a) It is negative, indicating healing. b) It results in immediate erythema. c) It is positive, indicating acantholysis. d) It shows no change in skin. Correct answer: It is positive, indicating acantholysis. What does a Tzanck Smear detect in Pemphigoid? a) Absence of eosinophils and presence of blisters. b) Presence of keratinocytes and cytoid bodies. c) Presence of eosinophils and absence of acantholytic cells. d) Only presence of acantholytic cells. Correct answer: Presence of eosinophils and absence of acantholytic cells. What is the characteristic skin lesion of Pemphigoid? a) Non-itchy macules on normal skin. b) Scaly lesions without edema. c) Blisters on dry skin without redness. d) Pruritic bullae on erythematous skin. Correct answer: Pruritic bullae on erythematous skin. What is acantholysis? a) The formation of new keratinocytes. b) The growth of epidermis layers. c) The adhesion of skin layers. d) The rupture or separation of desmosomes. Correct answer: The rupture or separation of desmosomes. Which structure is associated with adhesion in the epidermis? a) Desmosomes. b) Gap junctions. c) Tight junctions. d) Hemidesmosomes. Correct answer: Desmosomes. Which antibody is primarily associated with pemphigus foliaceous? a) Desmoglein-3 antibody. b) BPAG2 antibody. c) IgG antibody. d) Desmoglein-1 antibody. Correct answer: Desmoglein-1 antibody. Where does pemphigus foliaceous primarily affect? a) The dermis. b) The upper epidermis. c) The oral mucosa. d) The lower epidermis. Correct answer: The upper epidermis. What type of skin split is characterized in pemphigus vulgaris? a) Subcorneal split. b) Intraepidermal split. c) Basal split. d) Suprabasal split. Correct answer: Suprabasal split. In pemphigus vulgaris, oral lesions are present in what percentage of cases? a) About 50%. b) Approximately 95%. c) Only 20%. d) Always present. Correct answer: Approximately 95%. What is the Nikolsky sign used to determine? a) Separation of epidermal layers. b) Presence of keratinocytes. c) Depth of skin lesions. d) Function of desmosomes. Correct answer: Separation of epidermal layers. What is the Bulla Spread Sign indicative of? a) Healing of skin lesions. b) Formation of new blisters. c) Absence of skin lesions. d) Extension of bulla margins. Correct answer: Extension of bulla margins. What clinical feature is common with bullae in pemphigus? a) They are always painless. b) They are often itchy. c) They are very small. d) They heal with scarring. Correct answer: They are often itchy. What treatment is commonly used for localized pemphigus? a) Topical steroids. b) High dose oral antibiotics. c) Chemotherapy drugs. d) Radiation therapy. Correct answer: Topical steroids. What is Pemphigoid Gestationis also known as? a) Gestational psoriasis. b) Pregnancy-related eczema. c) Herpes Gestationis. d) Dermatitis herpetiformis. Correct answer: Herpes Gestationis. What clinical feature is typical of Pemphigoid Gestationis? a) Dry scaly patches. b) Hypopigmented spots. c) Acne-like lesions. d) Tense bullae. Correct answer: Tense bullae. What is the preferred treatment for Pemphigoid Gestationis? a) Topical antifungals. b) Systemic steroids. c) UV light therapy. d) Oral antihistamines. Correct answer: Systemic steroids. What are the characteristics of bullae? a) Painful, scabbing without lesions. b) Itchy, erosions due to scratching. c) Red, circular rashes on skin. d) Flaky, dry patches with no itching. Correct answer: Itchy, erosions due to scratching. What is the prognosis for bullae? a) Poor, requiring lengthy treatment. b) Uncertain and varies by age. c) Very good. d) Moderate, with potential complications. Correct answer: Very good. Which treatment options are available for bullae? a) Antibiotics and antifungal creams. b) Topical steroids and low dose oral steroids. c) Only systemic antibiotics. d) Moisturizers and topical antihistamines. Correct answer: Topical steroids and low dose oral steroids. What defines pemphigoid gestationis? a) A type of skin cancer. b) A viral skin infection. c) An allergic reaction to food. d) Bullous Pemphigoid occurring during pregnancy. Correct answer: Bullous Pemphigoid occurring during pregnancy. Which antigen is targeted in pemphigoid gestationis? a) Epidermal transglutaminase 3. b) BPAG2. c) IgA antibodies. d) Skin keratinocytes. Correct answer: BPAG2. Where do lesions typically occur in pemphigoid gestationis? a) Periumbilical region, thighs, abdomen. b) Face, palms, and soles. c) Arms, knees, and elbows. d) Back, neck, and scalp. Correct answer: Periumbilical region, thighs, abdomen. What is the drug of choice for pemphigoid gestationis? a) Antihistamines. b) Topical antifungals. c) Oral antibiotics. d) Systemic steroids. Correct answer: Systemic steroids. Which type of dermatosis affects children? a) Chronic Bullous Dermatosis of Childhood (CBDC). b) Pemphigus vulgaris. c) Linear IgA Disease (LAD). d) Dermatitis herpetiformis. Correct answer: Chronic Bullous Dermatosis of Childhood (CBDC). What is the autoantibody involved in CBDC? a) IgG. b) IgM. c) Autoantibody not specified. d) IgA. Correct answer: IgA. How are CBDC vesicles described? a) Linear streaks resembling scars. b) Annular arrangement resembling jewels. c) Red and inflamed patches. d) Flat lesions lacking pattern. Correct answer: Annular arrangement resembling jewels. What is the treatment for Chronic Bullous Dermatosis of Childhood? a) Oral steroids. b) Dapsone. c) Antihistamines. d) Moisturizing creams. Correct answer: Dapsone. What characterizes dermatitis herpetiformis? a) Chronically relapsing itchy immunobullous disease. b) Sudden onset of painful blisters. c) Temporary skin rash from sunlight. d) Non-itchy dry patches on skin. Correct answer: Chronically relapsing itchy immunobullous disease. Which target antigen is linked to dermatitis herpetiformis? a) BPAG2. b) IgA antibodies. c) Cytokeratin 1. d) Epidermal transglutaminase 3. Correct answer: Epidermal transglutaminase 3. What are common manifestations of dermatitis herpetiformis? a) Nausea and vomiting only. b) Malabsorption, diarrhea, abdominal pain. c) Fatigue and joint pain. d) Skin peeling and scaliness. Correct answer: Malabsorption, diarrhea, abdominal pain. What age group is most affected by dermatitis herpetiformis? a) Individuals aged 20-40 years. b) Children under 10 years. c) Elderly over 65 years. d) Teenagers aged 13-19. Correct answer: Individuals aged 20-40 years. What is the male-to-female ratio in dermatitis herpetiformis? a) 2:1. b) 1:1. c) 3:2. d) 4:1. Correct answer: 2:1. What are the primary lesions seen in dermatitis herpetiformis? a) Large, painful wounds. b) Flat, discolored patches. c) Red, non-itchy blisters. d) Extremely itchy papulovesicles. Correct answer: Extremely itchy papulovesicles. What is acantholysis? a) The rupture of desmosomes leading to skin blistering. b) The formation of new skin cells. c) The separation of keratinocytes from the dermis. d) The production of antibodies against skin structures. Correct answer: The rupture of desmosomes leading to skin blistering. What are desmosomes? a) Cells responsible for skin healing. b) Proteins that form the basement membrane. c) Structures providing adhesion between keratinocytes. d) Hormones regulating skin temperature. Correct answer: Structures providing adhesion between keratinocytes. Which antibody is primarily associated with pemphigus foliaceous? a) Desmoglein-3. b) Interleukin-6. c) Keratin-14. d) Desmoglein-1. Correct answer: Desmoglein-1. Pemphigus foliaceous affects which layer of the skin? a) Lower epidermis. b) Dermis. c) Hypodermis. d) Upper epidermis. Correct answer: Upper epidermis. What type of split is characterized by pemphigus vulgaris? a) Subcorneal split. b) Suprabasal split. c) Intraepidermal split. d) Subepidermal split. Correct answer: Suprabasal split. In which percentage of pemphigus vulgaris cases are oral lesions present? a) Approximately 95%. b) About 50%. c) Only 10%. d) Around 75%. Correct answer: Approximately 95%. How is the Nikolsky sign conducted? a) Apply direct pressure to the skin. b) Rub the skin with a cloth. c) Soak the skin in water. d) Apply tangential pressure over the skin. Correct answer: Apply tangential pressure over the skin. What result indicates a positive Nikolsky sign? a) Separation of upper from lower epidermis. b) Formation of new blisters. c) Increased skin redness. d) Immediate skin healing. Correct answer: Separation of upper from lower epidermis. What occurs during the bulla spread sign? a) Formation of new bullae. b) Marginal extension with an irregular border. c) Complete flattening of the bulla. d) Disappearance of the bulla. Correct answer: Marginal extension with an irregular border. What is the Asboe-Hansen sign? a) A test for keratinocyte proliferation. b) A type of skin biopsy. c) A variation of the bulla spread sign. d) An assessment of skin elasticity. Correct answer: A variation of the bulla spread sign. What does a Tzanck smear diagnose? a) Acantholytic cells. b) Bacterial infections. c) Fungal spores. d) Skin abrasions. Correct answer: Acantholytic cells. What does histopathology confirm? a) Skin cancer types. b) Allergic reactions. c) Diagnosis of pemphigus disorders. d) Scarring conditions. Correct answer: Diagnosis of pemphigus disorders. Where do grouped excoriations typically occur? a) Only on the face and neck b) On extensor surfaces and pressure points c) On the palms and soles d) Exclusively in flexor areas Correct answer: On extensor surfaces and pressure points What does a Tzanck smear primarily identify? a) Presence of eosinophils b) Presence of lymphocytes c) Presence of squamous cells d) Presence of neutrophils Correct answer: Presence of neutrophils Which finding is characteristic of histopathology in skin conditions? a) Necrosis of epidermis b) Vasculitis in dermis c) Papillary dermal fibrosis d) Subepidermal split observed Correct answer: Subepidermal split observed What pattern is observed in Direct Immunofluorescence for dermatitis herpetiformis? a) Granular pattern at BMZ b) Linear pattern in dermis c) Spotty pattern in epidermis d) Diffuse pattern throughout skin Correct answer: Granular pattern at BMZ Which autoantibody is associated with dermatitis herpetiformis? a) IgA b) IgM c) IgG d) IgE Correct answer: IgA What is the dietary recommendation for dermatitis herpetiformis? a) High-protein diet b) Dairy-free diet c) Low-sugar diet d) Gluten-Free Diet Correct answer: Gluten-Free Diet What is the drug of choice for treating dermatitis herpetiformis? a) Prednisone b) Dapsone c) Methotrexate d) Sulfasalazine Correct answer: Dapsone How do mild mechanical injuries affect mechanobullous disorders? a) Lead to blisters or bullae formation b) Cause skin thickening c) Result in pigmentation changes d) Induce chronic inflammation Correct answer: Lead to blisters or bullae formation What genetic mutations are involved in EB Simplex? a) K5/K14 mutations b) Laminin gene mutations c) Collagen 7 mutations d) Keratin 17 mutations Correct answer: K5/K14 mutations Which family history clue may indicate a predisposition to mechanobullous disorders? a) History of allergies b) High cholesterol levels c) No family history d) Consanguineous marriage Correct answer: Consanguineous marriage What result is typically expected from DIF in mechanobullous disorders? a) Negative results b) Positive findings for IgG c) Positive findings for IgA d) Weakly positive for complement Correct answer: Negative results What is considered the definitive treatment for epidermolysis bullosa? a) Topical corticosteroids b) Antibiotic therapy c) Phototherapy d) Gene therapy Correct answer: Gene therapy What additional care is necessary for individuals with mechanobullous disorders? a) Aggressive surgery b) Supportive management c) Radiation therapy d) Chemotherapy Correct answer: Supportive management What type of inheritance pattern does Darier's Disease follow? a) Autosomal Recessive (AR) b) X-Linked Dominant c) Autosomal Dominant (AD) d) X-Linked Recessive Correct answer: Autosomal Dominant (AD) Which gene defect is associated with Hailey-Hailey Disease? a) ATP2A2 b) COL7A1 c) P53 d) ATP2C1 Correct answer: ATP2C1 Which chromosome is linked to Darier's Disease? a) Chromosome 12 b) Chromosome 14 c) Chromosome 3 d) Chromosome 7 Correct answer: Chromosome 3 What is the definition of alopecia? a) Loss of hair b) Inflammation of the scalp c) Baldness due to genetics d) Hair thinning with age Correct answer: Loss of hair What characterizes cicatricial alopecia? a) Stress-related hair loss b) Damage to stem cells in the bulge area c) Genetic predisposition d) Hormonal changes Correct answer: Damage to stem cells in the bulge area Which type of alopecia is characterized by patchy hair loss? a) Androgenetic Alopecia b) Alopecia Areata c) Telogen Effluvium d) Tinea Capitis Correct answer: Alopecia Areata What is one condition classified under Non-Cicatricial Alopecia? a) Trichotillomania b) Lichen Planopilaris c) Frontal Fibrosing Alopecia d) Androgenetic Alopecia Correct answer: Trichotillomania What triggers Anagen Effluvium? a) Stress and infections b) Genetic factors c) Hormonal imbalances d) Chemotherapy and radiation Correct answer: Chemotherapy and radiation Which is a common symptom of Alopecia Areata? a) Inflamed scalp b) Gradual thinning of hair c) No hair growth cycle d) Circular areas of complete hair loss Correct answer: Circular areas of complete hair loss What appearance do lesions exhibit in Alopecia Areata? a) Frayed Hair Ends b) Split Ends c) Exclamation Mark Hair d) Brittle Hair Strands Correct answer: Exclamation Mark Hair What histopathological finding is indicative of Alopecia Areata? a) Follicular keratosis b) Sebaceous gland hyperplasia c) Vascular proliferation d) Peribulbar lymphocytic infiltrates Correct answer: Peribulbar lymphocytic infiltrates What does 'Swarm of bees' appearance indicate? a) Seborrheic dermatitis b) Lymphocytic infiltrates around hair follicles c) Follicular scarring d) Bacterial infection Correct answer: Lymphocytic infiltrates around hair follicles What nail condition is associated with Alopecia Areata? a) Onycholysis b) Subungual hematoma c) Fungal infection d) Nail Pitting with geometric patterns Correct answer: Nail Pitting with geometric patterns What is the most potent topical steroid? a) Triamcinolone Acetonide b) Clobetasol c) Hydrocortisone d) Betamethasone Correct answer: Clobetasol Which steroid is used for extensive disease management? a) Topical Steroids b) Oral Antihistamines c) Systemic Steroids d) Bacterial Antibiotics Correct answer: Systemic Steroids What characterizes Trichotillomania? a) Complete baldness b) Hair follicle infection c) Compulsive pulling out of hair d) Hair growth stimulation Correct answer: Compulsive pulling out of hair What is a symptom of Trichotillomania? a) Long hair growth b) Incomplete hair loss in patches c) Total hair thickness d) Uniform hair length Correct answer: Incomplete hair loss in patches What is Male Pattern Androgenetic Alopecia marked by? a) Full hair coverage b) Fronto-temporal recession c) Hair growing on cheeks d) Hair growing on palms Correct answer: Fronto-temporal recession What distinguishes Female Pattern Androgenetic Alopecia? a) Only affects frontal hairline b) Identical to male pattern c) Similar patterns with different distribution d) Causes scarring Correct answer: Similar patterns with different distribution What condition leads to permanent hair loss through scarring? a) Male Pattern Alopecia b) Scarring Alopecia c) Lichen Planopilaris d) Telogen Effluvium Correct answer: Scarring Alopecia Which disorder is a type of scarring alopecia? a) Androgenetic Alopecia b) Lichen Planopilaris (LPP) c) Tinea Capitis d) Miliaria Correct answer: Lichen Planopilaris (LPP) What systemic condition may have skin manifestations? a) Systemic Lupus Erythematosus b) Scarring Alopecia c) Eccrine Glands Disorder d) Trichotillomania Correct answer: Systemic Lupus Erythematosus What type of infection is Tinea Capitis? a) Bacterial skin infection b) Viral hair loss c) Fungal skin allergy d) An inflammatory fungal infection Correct answer: An inflammatory fungal infection Where are eccrine glands primarily located? a) Throughout the body, especially on palms and soles b) Only under the arms c) Around the scalp d) In the groin area Correct answer: Throughout the body, especially on palms and soles What signifies the function of apocrine glands? a) Sweat production b) Thermoregulation c) Pheromone production and body odor d) Heat rash formation Correct answer: Pheromone production and body odor What condition results from blockage of eccrine glands? a) Scarring Alopecia b) Miliaria (heat rash) c) Tinea Capitis d) Alopecia Areata Correct answer: Miliaria (heat rash) What causes Fordyce Disease? a) Excessive sun exposure. b) Allergic reactions to cosmetics. c) Bacterial infections in skin. d) Obstruction of the apocrine duct. Correct answer: Obstruction of the apocrine duct. Which area is NOT commonly affected by Fordyce Disease? a) Axilla. b) Upper lip. c) Areola of the nipple. d) Feet. Correct answer: Feet. What do Fordyce spots primarily resemble? a) Red inflamed bumps. b) Black comedones. c) Yellow micro papules. d) Whiteheads. Correct answer: Yellow micro papules. What is the main organism responsible for Acne Vulgaris? a) Staphylococcus aureus. b) Escherichia coli. c) Cutibacterium acnes. d) Propionibacterium granulosum. Correct answer: Cutibacterium acnes. Which grade of Acne Vulgaris includes only comedones? a) Grade 2. b) Grade 1. c) Grade 3. d) Grade 4. Correct answer: Grade 1. What are open comedones commonly known as? a) Whiteheads. b) Yellow papules. c) Red pustules. d) Blackheads. Correct answer: Blackheads. What is a common side effect of Isotretinoin? a) Excessive sweating. b) Increased hair growth. c) Cheilitis (dryness of lips). d) Oily skin. Correct answer: Cheilitis (dryness of lips). Which topical treatment is a Vitamin A analogue? a) Benzoyl peroxide. b) Isotretinoin. c) Tretinoin. d) Salicylic acid. Correct answer: Tretinoin. What dosage range is common for Isotretinoin treatment? a) 0.5-1 mg/kg daily. b) 2-3 mg/kg weekly. c) 1-2 mg/kg monthly. d) 0.2-0.5 mg/kg daily. Correct answer: 0.5-1 mg/kg daily. What are closed comedones commonly referred to as? a) Blackheads. b) Whiteheads. c) Cystic acne. d) Papules. Correct answer: Whiteheads. What are topical retinoids primarily used for? a) Dry skin treatment. b) Acne treatment. c) Anti-aging solutions. d) Sunburn relief. Correct answer: Acne treatment. Which of the following is NOT a topical retinoid? a) Adapalene. b) Isotretinoin. c) Tretinoin. d) Retinaldehyde. Correct answer: Isotretinoin. What characterizes Grade 2 Acne Vulgaris? a) Severe pustular lesions. b) Predominantly red papular lesions. c) Nodules with deep scarring. d) Mild comedonal acne. Correct answer: Predominantly red papular lesions. What is the main characteristic of Grade 4 Acne Vulgaris? a) Pustular lesions. b) Nodulocystic acne with scars. c) Mild papules. d) Only comedones present. Correct answer: Nodulocystic acne with scars. Which treatment is considered first-line for nodulocystic acne? a) Isotretinoin. b) Topical benzoyl peroxide. c) Oral antibiotics. d) Salicylic acid peels. Correct answer: Isotretinoin. What is a common side effect of isotretinoin treatment? a) Increased appetite. b) Cheilitis (dryness of lips). c) Excessive sweating. d) Oily skin. Correct answer: Cheilitis (dryness of lips). What is the recommended dosage range for isotretinoin? a) 1-2 mg/kg daily. b) 0.1-0.5 mg/kg daily. c) 0.5-1 mg/kg daily. d) 2-3 mg/kg daily. Correct answer: 0.5-1 mg/kg daily. What lab monitoring should be done during oral retinoid treatment? a) Liver function tests. b) Thyroid function tests. c) Complete blood count. d) Blood pressure assessment. Correct answer: Liver function tests. What is SAPHO syndrome associated with? a) Fatigue, Headaches, Skin rash. b) Synovitis, Acne, Pustulosis, Hyperostosis, Osteitis. c) High blood pressure. d) Fever, Cough, Loss of taste. Correct answer: Synovitis, Acne, Pustulosis, Hyperostosis, Osteitis. Which syndrome includes Acne and Pyogenic Arthritis? a) PAPA Syndrome. b) SAPHO Syndrome. c) HAIRAN Syndrome. d) SAHA Syndrome. Correct answer: PAPA Syndrome. In acne, where are comedones typically present? a) Palms and soles. b) Scalp and neck. c) Just around the mouth. d) Face, upper trunk, chest, shoulders. Correct answer: Face, upper trunk, chest, shoulders. What distinguishes Rosacea from Acne? a) Rosacea has only pustules. b) Comedones absent in Rosacea. c) Acne does not cause flushing. d) Rosacea occurs exclusively on the back. Correct answer: Comedones absent in Rosacea. What triggers Rosacea symptoms? a) Excessive lubrication. b) Cold weather. c) Frequent hand washing. d) Sunlight. Correct answer: Sunlight. What is a hallmark symptom of Rosacea? a) Severe dry patches. b) Abnormal vascular reactivity leading to flushing. c) Rapid hair growth. d) Persistent itching. Correct answer: Abnormal vascular reactivity leading to flushing. What causes Fordyce disease? a) Excessive sweating of the skin. b) Blockage of hair follicles. c) Obstruction of the apocrine duct. d) Hormonal imbalance in the body. Correct answer: Obstruction of the apocrine duct. What are the characteristics of Fordyce spots? a) Yellow micro papules. b) Red inflamed lesions. c) Crusty, scaly patches. d) Large, dark moles. Correct answer: Yellow micro papules. Which areas are commonly affected by Fordyce disease? a) Feet and hands. b) Scalp and face. c) Back and abdomen. d) Axilla and areola of the nipple. Correct answer: Axilla and areola of the nipple. What is the primary cause of Acne Vulgaris? a) Staphylococcus aureus. b) Escherichia coli. c) Candida albicans. d) Cutibacterium acnes. Correct answer: Cutibacterium acnes. What type of lesions are characteristic of Acne Vulgaris? a) Fluid-filled blisters. b) Flat, discolored patches. c) Polymorphic cutaneous lesions. d) Raised pustules with fever. Correct answer: Polymorphic cutaneous lesions. What are open comedones often described as? a) Red and inflamed bumps. b) Black due to oxidation of sebum. c) White in appearance. d) Dark, scabby spots. Correct answer: Black due to oxidation of sebum. What is the classification for mild Acne Vulgaris? a) Grade 2: Moderate. b) Grade 1: Mild form. c) Grade 3: Severe. d) Grade 4: Cystic. Correct answer: Grade 1: Mild form. Where else can Montgomery tubercles be found? a) On the scalp. b) Around the navel. c) On the toes. d) In the breast. Correct answer: In the breast. Which glands are involved in Fordyce disease in the eyelids? a) Sweat glands and salivary glands. b) Meibomian glands and glands of Zeis. c) Sebaceous glands and thyroid glands. d) Adrenal glands and pituitary glands. Correct answer: Meibomian glands and glands of Zeis. What skin condition is characterized by closed and open comedones? a) Fordyce Disease. b) Psoriasis. c) Acne Vulgaris. d) Eczema. Correct answer: Acne Vulgaris. What is a characteristic of the papulopustular stage? a) Presence of papules and pustules. b) Presence of nodules and plaques. c) Absence of visible lesions. d) Thickening of the skin. Correct answer: Presence of papules and pustules. What is a key feature of the phymatous type? a) Skin over the nose becomes thick. b) Skin over the forehead becomes thin. c) Skin remains smooth and unthickened. d) Skin on the cheeks becomes inflamed. Correct answer: Skin over the nose becomes thick. Which treatment is considered the treatment of choice for phymatous type? a) Topical antibiotics. b) CO2 laser excision followed by resurfacing. c) Oral retinoids. d) Cryotherapy. Correct answer: CO2 laser excision followed by resurfacing. What describes rhinophyma? a) Thickening over the chin. b) Thickening over the ear. c) Thickening over the nose. d) Thickening over the forehead. Correct answer: Thickening over the nose. How does rosacea differ from systemic lupus erythematosus in terms of photosensitivity? a) Rosacea: Present; SLE: Absent. b) Both have photosensitivity. c) Neither condition has photosensitivity. d) Rosacea: Absent; SLE: Present. Correct answer: Rosacea: Absent; SLE: Present. What is an absent symptom in rosacea compared to SLE? a) Erythema. b) Malar rash. c) Papules and pustules. d) Photosensitivity. Correct answer: Malar rash. Which trigger is associated with rosacea? a) Cold weather. b) Spicy food. c) Sun exposure. d) Fatty foods. Correct answer: Spicy food. What systemic features does SLE exhibit that rosacea does not? a) Transient erythema. b) Fever, arthralgia, myalgia, oral ulcers. c) Presence of papules. d) Photosensitivity. Correct answer: Fever, arthralgia, myalgia, oral ulcers. What type of leukonychia is associated with chronic arsenic poisoning? a) Apparent leukonychia. b) Half & half nails. c) True leukonychia. d) Terry's nails. Correct answer: True leukonychia. What are Mees lines? a) Horizontal white bands on the nail plate. b) Vertical ridges on the nail. c) Half-white, half-pink nails. d) Abnormal pink band at the tip. Correct answer: Horizontal white bands on the nail plate. Terry's nails are characterized by which feature? a) White nails with a narrow pink band at the tip. b) Purely pink nails. c) Half-white, half-pink appearance. d) Horizontal white bands on the nail. Correct answer: White nails with a narrow pink band at the tip. What are Muehrcke's Bands? a) Pale nails with no bands b) Curved nails with various colors c) Ridges on nails with no color change d) Paired transverse white bands on nails. Correct answer: Paired transverse white bands on nails. Which condition is associated with Muehrcke's Bands? a) Diabetes Mellitus b) Psoriasis c) Eczema d) Hypoalbuminemia Correct answer: Hypoalbuminemia What characterizes Half & Half Nails? a) Uniform color throughout the nail b) Half black and half white c) All white with no color variation d) Proximal 50% white and distal 50% brown to pink. Correct answer: Proximal 50% white and distal 50% brown to pink. Half & Half Nails are associated with which condition? a) Heart Disease b) Pulmonary Hypertension c) Cirrhosis d) Kidney Failure Correct answer: Kidney Failure What defines Terry's Nails? a) Proximal 90% white and distal 10% pink to brown. b) All brown with white tips c) Clear with no coloration d) Uniform pink all over Correct answer: Proximal 90% white and distal 10% pink to brown. Terry's Nails are indicative of which illness? a) Renal Failure b) Lung Cancer c) Liver Disease d) Thyroid Disorder Correct answer: Liver Disease What is Melasma characterized by? a) Symmetrical hyperpigmented brownish macules. b) Red inflamed patches on skin c) Eruptions of small blisters d) Flaky skin with white spots Correct answer: Symmetrical hyperpigmented brownish macules. In which areas is Melasma commonly found? a) Feet and hands b) Upper back and arms c) Palms and soles d) Nose, malar area, and mandibular area. Correct answer: Nose, malar area, and mandibular area. How do freckles and lentigines differ in melanocyte activity? a) Freckles have decreased, lentigines normal b) Freckles have normal melanocyte number, lentigines increased. c) Both have increased activity d) Both have normal activity Correct answer: Freckles have normal melanocyte number, lentigines increased. Where are freckles typically located? a) Protected areas only b) Sun-exposed areas. c) Anywhere on the body d) Only on the face and neck Correct answer: Sun-exposed areas. Which condition worsens in the summer? a) Lentigines b) Freckles c) Terry's Nails d) Lupus Correct answer: Freckles What genetic condition increases sensitivity to UV light? a) Ehler-Danlos Syndrome b) Marfan Syndrome c) Xeroderma Pigmentosum d) Turner Syndrome Correct answer: Xeroderma Pigmentosum Peutz-Jeghers Syndrome is inherited in which manner? a) Autosomal Recessive b) X-linked Dominant c) Autosomal Dominant d) Not hereditary Correct answer: Autosomal Dominant What are the key features of Peutz-Jeghers Syndrome? a) Mucosal lentigines and gastrointestinal polyps. b) Skin rash and liver spots c) Bone growths and joint pain d) Blood vessel malformations and cysts Correct answer: Mucosal lentigines and gastrointestinal polyps. What is the most common type of polyp? a) Becker's Nevus b) Nevus of Ota c) Mongolian Spot d) Hamartomatous Polyps Correct answer: Hamartomatous Polyps What characterizes Becker's Nevus? a) Congenital nevus found at birth. b) Flat and smooth skin appearance. c) Always found bilaterally on limbs. d) Acquired nevus typically seen during puberty. Correct answer: Acquired nevus typically seen during puberty. Where is Becker's Nevus commonly located? a) Lower back and thighs. b) Chest and upper shoulder. c) Face and neck. d) Forearms and legs. Correct answer: Chest and upper shoulder. What does dermal melanocytosis produce? a) Decrease in skin pigmentation. b) Proliferation of melanocytes in the dermis. c) Absence of melanocytes in the epidermis. d) Overgrowth of skin cells. Correct answer: Proliferation of melanocytes in the dermis. What causes the blue color in dermal melanocytosis? a) The Tyndall effect. b) Direct sunlight exposure. c) Low melanin production. d) Overproduction of collagen. Correct answer: The Tyndall effect. What describes the appearance of ceruloderma? a) Redness with bumps. b) Yellowish skin tone. c) Blue to slate grey skin color. d) White patches on skin. Correct answer: Blue to slate grey skin color. Where is a Mongolian spot commonly found? a) Forehead and cheeks. b) Lumbo-sacral region in infants. c) Chest and arms. d) Hands and feet. Correct answer: Lumbo-sacral region in infants. How does a Mongolian spot resolve? a) Requires laser treatment. b) Only resolves with medication. c) Needs surgery. d) Resolves spontaneously. Correct answer: Resolves spontaneously. What is a key characteristic of Nevus of Ota? a) Bilateral bright red spots. b) Flat, yellowish patches. c) Unilateral, blue to slate grey color. d) Brown raised lesions. Correct answer: Unilateral, blue to slate grey color. Which nerve divisions does Nevus of Ota follow? a) 1st and 2nd divisions of the Trigeminal nerve. b) Facial and glossopharyngeal nerves. c) Vagus nerve and cervical nerves. d) Musculocutaneous and radial nerves. Correct answer: 1st and 2nd divisions of the Trigeminal nerve. Where is Nevus of Ota primarily located? a) Primarily on the face. b) Palms and soles. c) Scalp and forehead. d) Knees and elbows. Correct answer: Primarily on the face. What characterizes Nevus of Ito? a) Flat and smooth texture. b) Only found on the limbs. c) Always bilateral. d) Similar to Nevus of Ota but differently located. Correct answer: Similar to Nevus of Ota but differently located. Where does Nevus of Ito follow? a) Anterior supraclavicular nerve. b) Thoracic and lumbar regions. c) Cervical spine and upper back. d) Posterior supraclavicular and lateral brachial nerves. Correct answer: Posterior supraclavicular and lateral brachial nerves. What is Nevus of Hori? a) A type of Mongolian spot. b) Bilateral Nevus of Ota. c) Unilateral nevus on the back. d) A congenital skin condition. Correct answer: Bilateral Nevus of Ota. Which treatment is used for Nevus of Ota/Ito? a) Cryotherapy. b) Topical steroids. c) Microneedling. d) Q-switched Nd:YAG laser. Correct answer: Q-switched Nd:YAG laser. What type of disorder is albinism? a) Acquired skin condition. b) Autoimmune skin disorder. c) Bacterial skin infection. d) Congenital disorder of hypo depigmentation. Correct answer: Congenital disorder of hypo depigmentation. What is the biochemical defect in albinism? a) Excessive melanin production. b) Malfunctioning vitamin D synthesis. c) Defective tyrosinase enzyme. d) Low collagen levels. Correct answer: Defective tyrosinase enzyme. What is ocular albinism? a) A form of albinism limited to the eyes. b) Involves skin and hair as well. c) A condition with white patches on skin. d) Characterized by the absence of eyes. Correct answer: A form of albinism limited to the eyes. What is a major characteristic of piebaldism? a) Complete absence of hair. b) White patches all over the body. c) Presence of a white forelock. d) Vasoconstriction of blood vessels. Correct answer: Presence of a white forelock. What inheritance pattern is associated with piebaldism? a) X-linked recessive inheritance. b) Autosomal recessive inheritance. c) Autosomal dominant inheritance. d) Mitochondrial inheritance pattern. Correct answer: Autosomal dominant inheritance. What defect leads to nevus depigmentosus? a) Defect in the transfer of melanosomes. b) Complete absence of melanin synthesis. c) Damage to the keratinocytes directly. d) Deficiency in blood vessel growth. Correct answer: Defect in the transfer of melanosomes. When does nevus depigmentosus typically onset? a) During adolescence. b) In early childhood. c) In late adulthood. d) At birth. Correct answer: At birth. What are vitiligo's most accepted causes? a) Genetic mutation in keratinocytes. b) Auto-antibodies destroy melanocytes. c) Environmental exposure to toxins. d) Viral infection targeting skin cells. Correct answer: Auto-antibodies destroy melanocytes. What is a characteristic of segmental vitiligo? a) Bilateral, symmetrical patches. b) Completely depigmented appearance. c) Appears only on the face. d) Unilateral, does not cross the midline. Correct answer: Unilateral, does not cross the midline. What treatments are available for segmental vitiligo? a) Melanocyte transplanting and skin grafting. b) Dietary supplements and topical steroids. c) Laser therapy and chemical peels. d) Oral antihistamines and pain relief. Correct answer: Melanocyte transplanting and skin grafting. What links vitiligo with other conditions? a) Only linked with skin diseases. b) Associated with aging and sun exposure. c) Conditions like thyroid dysfunction and T1DM. d) Only occurs in individuals with allergies. Correct answer: Conditions like thyroid dysfunction and T1DM. What is a defining feature of nevus anemicus? a) A genetic mutation in skin cells. b) A virus causing discoloration. c) An autoimmune response to pigments. d) Involves a pharmacological defect. Correct answer: Involves a pharmacological defect. What areas does Acrofacial Vitiligo affect? a) The face, head, and distal extremities. b) Only the torso and upper limbs. c) Entire body except for limbs. d) Primarily the back and shoulders. Correct answer: The face, head, and distal extremities. Which type of vitiligo is the most common? a) Vitiligo Vulgaris. b) Mucosal Vitiligo. c) Universal Vitiligo. d) Focal Vitiligo. Correct answer: Vitiligo Vulgaris. What characterizes Vitiligo Vulgaris? a) Randomly distributed dark patches. b) Only affects the scalp. c) Large, continuous areas of depigmentation. d) Symmetrically distributed depigmented macular lesions. Correct answer: Symmetrically distributed depigmented macular lesions. What is the extent of Universal Vitiligo? a) 10-20% of body surface area involved. b) 50-70% of body surface area affected. c) Full body without exception. d) 80-90% of body surface area depigmented. Correct answer: 80-90% of body surface area depigmented. What type of vitiligo has small isolated lesions? a) Mucosal Vitiligo. b) Focal Vitiligo. c) Universal Vitiligo. d) Leucotrichia. Correct answer: Focal Vitiligo. What is Koebner's Phenomenon associated with? a) Lesions fading with time. b) Increased pigmentation in adjacent areas. c) Random distribution of new lesions. d) New lesions appear at the site of trauma. Correct answer: New lesions appear at the site of trauma. What condition is indicated by Leucotrichia? a) Increased hair loss. b) Dark hair pigmentation. c) Hair turning white due to vitiligo. d) Hair becoming curly. Correct answer: Hair turning white due to vitiligo. What does Exaggerated White Fluorescence indicate? a) Excessive melanin production. b) Presence of collagen in the dermis. c) Absence of skin lesions. d) Inflammatory skin reaction. Correct answer: Presence of collagen in the dermis. For less than 20% body surface area involvement, what is the treatment? a) Topical steroids and tacrolimus. b) Systemic steroids only. c) NBUVB therapy only. d) Surgical skin grafts. Correct answer: Topical steroids and tacrolimus. What treatment is used when more than 20% BSA is involved? a) Only topical creams. b) Antihistamines for itching. c) Systemic steroids and Azathioprine. d) Laser treatments. Correct answer: Systemic steroids and Azathioprine. What does Chemical Leukoderma indicate? a) Increased melanin production. b) Skin infection leading to color change. c) Allergic reaction to skin products. d) Melanocyte destruction due to chemicals. Correct answer: Melanocyte destruction due to chemicals. What chemical causes Bindi Dermatitis? a) Para tertiary butyl phenol (PTBP) glue. b) Benzoyl peroxide. c) Coconut oil. d) Alcohol-based solutions. Correct answer: Para tertiary butyl phenol (PTBP) glue. What is Mono Benzyl Ether of Hydroquinone (MBEH) linked to? a) Increased skin moisture. b) Enhancement of skin tone. c) Antimicrobial properties for skin. d) Destruction of melanocytes from rubber footwear. Correct answer: Destruction of melanocytes from rubber footwear. What is psoriasis? a) A chronic, T-cell mediated inflammatory disease. b) A viral skin infection. c) A bacterial condition. d) A genetic disorder only. Correct answer: A chronic, T-cell mediated inflammatory disease. What characterizes the skin features of psoriasis? a) Red bumps with yellow crust. b) Erythematous plaques with silvery white scaling. c) Dry patches with blisters. d) Dark spots with no scaling. Correct answer: Erythematous plaques with silvery white scaling. Which joint condition is associated with psoriasis? a) Osteoporosis. b) Arthritis. c) Rheumatism. d) Gout. Correct answer: Arthritis. What condition is NOT part of metabolic syndrome associated with psoriasis? a) Low cholesterol levels. b) Hyperlipidemia. c) Hypertension. d) Visceral obesity. Correct answer: Low cholesterol levels. When do psoriasis symptoms tend to worsen? a) During summer. b) After rain. c) During winter. d) In hot climates. Correct answer: During winter. Which infection is a notable trigger for guttate psoriasis? a) Viral infections. b) Streptococcal infections. c) Fungal infections. d) HIV only. Correct answer: Streptococcal infections. What role does substance abuse play in psoriasis? a) It cures the disease. b) It can contribute to flare-ups. c) It has no effect. d) It reduces inflammation. Correct answer: It can contribute to flare-ups. What is epidermal hyperproliferation in psoriasis? a) An increase in keratinocyte production. b) Decreased skin turnover. c) Reduced skin hydration. d) Increased skin elasticity. Correct answer: An increase in keratinocyte production. Which cytokine is considered a Th1 cytokine? a) IL-17. b) IL-2. c) IL-22. d) TNF beta. Correct answer: IL-2. What are the key features of Chronic Plaque Psoriasis? a) Fluid-filled blisters. b) Only flat lesions. c) Erythematous scaly papules and plaques. d) Bright red rashes. Correct answer: Erythematous scaly papules and plaques. What defines pustular psoriasis? a) Constantly itchy patches. b) Presence of sterile pustules. c) Deep cracks in the skin. d) Bleeding sores. Correct answer: Presence of sterile pustules. What is the acute form of generalized pustular psoriasis called? a) Von Zumbusch Type. b) Mild pustular type. c) Chronic variant. d) Papular type. Correct answer: Von Zumbusch Type. What is a common misnomer for Pustular Psoriasis of Pregnancy? a) Impetigo Herpetiformis. b) Psoriasis vulgaris. c) Follicular psoriasis. d) Chronic disease type. Correct answer: Impetigo Herpetiformis. What characterizes erythrodermic psoriasis? a) >50% body surface area with scaling b) >90% body surface area with erythema c) >90% body surface area with no symptoms d) >75% body surface area with dryness Correct answer: >90% body surface area with erythema Which condition can lead to erythroderma? a) Psoriatic arthritis b) Rheumatoid arthritis c) Osteoarthritis d) Gout Correct answer: Psoriatic arthritis What type of joint involvement is common in classical psoriasis? a) Distal Interphalangeal (DIP) joints b) Shoulder joints c) Knee joints d) Spinal joints Correct answer: Distal Interphalangeal (DIP) joints Which type of arthritis is considered the worst in psoriasis? a) Psoriatic arthritis b) Seronegative arthritis c) Erosive arthritis d) Arthritis mutilans Correct answer: Arthritis mutilans What does dactylitis refer to? a) Inflammation of the digits b) Pain in the spine c) Swelling of the knees d) Numbness in the fingers Correct answer: Inflammation of the digits What does the Auspitz sign indicate? a) Heavy scaling of the skin b) Complete absence of inflammation c) Pinpoint bleeding spots upon scraping d) Dark pigmentation of lesions Correct answer: Pinpoint bleeding spots upon scraping What is the Grattage test used to identify? a) Koebners phenomenon b) Dactylitis c) Arthritis mutilans d) Auspitz sign Correct answer: Auspitz sign What is the clinical significance of Koebners phenomenon? a) Shows complete disease remission b) Reveals new treatment effectiveness c) Indicates active disease; avoid trauma d) Indicates no skin involvement Correct answer: Indicates active disease; avoid trauma Which is a true form of Koebners phenomenon? a) Lesions appearing spontaneously b) Disappearance of lesions after treatment c) Lesions that occur in different locations d) Appearance of lesions after trauma Correct answer: Appearance of lesions after trauma Which condition is NOT a rare cause of Koebners phenomenon? a) Kaposi sarcoma b) Psoriasis c) Lichen planus d) Viral warts Correct answer: Psoriasis Which rare condition can cause Koebners phenomenon? a) Rheumatoid arthritis b) Psoriatic arthritis c) Osteoporosis d) Darier disease Correct answer: Darier disease What can result from trauma in patients with psoriasis? a) New lesions due to Koebners phenomenon b) Improvement of existing lesions c) No change in skin condition d) Reduction in inflammation Correct answer: New lesions due to Koebners phenomenon What characterizes nail psoriasis? a) Changes in nail appearance and structure. b) Exclusively nail length abnormalities. c) Only discoloration of the nail plate. d) Severe pain in the nail area. Correct answer: Changes in nail appearance and structure. What does pitting in nails indicate? a) An infection in the nail bed. b) A sign of normal nail growth. c) A result of poor nail hygiene. d) A significant clinical change in psoriasis. Correct answer: A significant clinical change in psoriasis. What is the oil drop sign in nail psoriasis? a) Red spots on the nail surface. b) Cracking of the nail's edge. c) Yellow discoloration of the nail bed. d) A bluish tint to the nail. Correct answer: Yellow discoloration of the nail bed. What does leukonychia refer to? a) Dark spots on the nail. b) White discoloration of the nail plate. c) A condition of nail thickening. d) The splitting of the nail layers. Correct answer: White discoloration of the nail plate. What is onycholysis? a) Nail bed infection. b) Rapid nail growth. c) A type of nail polish. d) Separation of the nail plate from the bed. Correct answer: Separation of the nail plate from the bed. What indicates subungual hyperkeratosis? a) Nail discoloration. b) Visible nail growth patterns. c) Accumulation of material below the nail plate. d) Splitting of the nail edges. Correct answer: Accumulation of material below the nail plate. What are splinter hemorrhages? a) Skin tags around the nail. b) White spots on the nail surface. c) Bumps along the nail bed. d) Small, linear blood spots under the nail. Correct answer: Small, linear blood spots under the nail. Which type of nail change is most common in psoriasis? a) Onycholysis. b) Subungual hyperkeratosis. c) Pitting of nails. d) Leukonychia. Correct answer: Pitting of nails. What distinguishes DIC pitting from other nail changes? a) It involves pain management. b) It's associated with acute infections. c) It is distinct from changes seen in Alopecia Areata. d) It resolves without treatment. Correct answer: It is distinct from changes seen in Alopecia Areata. What does the hypopigmented rim indicate? a) Abnormal prostaglandin levels in lesions. b) A healing process of the nail. c) Normal pigmentation of the skin. d) Common allergic reactions. Correct answer: Abnormal prostaglandin levels in lesions. What characterizes Scalp Psoriasis? a) Red patches with dry skin scabs. b) Thick micaceous scales without alopecia. c) Dry, flaky skin with hair loss. d) Smooth, shiny surface without scales. Correct answer: Thick micaceous scales without alopecia. Which type of psoriasis is notable for the absence of scaling? a) Inverse Flexural Psoriasis. b) Scalp Psoriasis. c) Plaque Psoriasis. d) Guttate Psoriasis. Correct answer: Inverse Flexural Psoriasis. What does hyperkeratosis indicate in psoriasis? a) Reduction in skin cell production. b) Increase in skin moisture levels. c) Inflammation of dermal layers. d) Pathological thickening of the stratum corneum. Correct answer: Pathological thickening of the stratum corneum. What does parakeratosis reflect in skin changes? a) Decrease in skin elasticity. b) Retention of nuclei in the topmost layer. c) Absence of normal skin cells. d) Formation of papules and nodules. Correct answer: Retention of nuclei in the topmost layer. What is a key feature of the stratum granulosum in psoriasis? a) Absent/decreased granular layer. b) Thickening of the skin barrier. c) Abnormal pigmentation in skin layers. d) Increased moisture retention. Correct answer: Absent/decreased granular layer. What are Munroe's microabscesses indicative of? a) Presence of neutrophils in the stratum corneum. b) Skin dehydration and cracking. c) Malfunction of hair follicles. d) Increase in melanin production. Correct answer: Presence of neutrophils in the stratum corneum. Where are spongiform pustules of Kogoj typically found? a) Stratum corneum. b) Stratum granulosum. c) Stratum spinosum. d) Dermal papillae layer. Correct answer: Stratum spinosum. What does supra papillary thinning result from? a) Increase in dermal adipose tissue. b) Thickening of the stratum spinosum. c) Destruction of the basal layer. d) Decrease in blood circulation. Correct answer: Thickening of the stratum spinosum. Which vascular change is associated with psoriasis lesions? a) Narrowed and constricted blood vessels. b) Normal capillary structure. c) Atherosclerotic vessel changes. d) Dilated and tortuous dermal vessels. Correct answer: Dilated and tortuous dermal vessels. How is treatment for psoriasis assessed based on body surface area? a) BSA > 20% for topical treatments. b) BSA < 5% for systemic treatments. c) BSA < 10% for topical treatments. d) BSA < 15% for phototherapy. Correct answer: BSA < 10% for topical treatments. What is the treatment recommendation for BSA > 10%? a) Phototherapy is suggested. b) Oral medications should be prioritized. c) No treatment is required. d) Only topical creams are recommended. Correct answer: Phototherapy is suggested. What does parakeratosis indicate in psoriasis histopathology? a) Absence of the stratum granulosum. b) Thickening of the stratum spinosum. c) Retention of nuclei in the topmost skin layer. d) Presence of dilated dermal vessels. Correct answer: Retention of nuclei in the topmost skin layer. What is a characteristic feature of agranulosis? a) Thickening of the stratum spinosum. b) Retention of nuclei in the epidermis. c) Presence of T-cell infiltration. d) Absence of the stratum granulosum. Correct answer: Absence of the stratum granulosum. What does acanthosis refer to in psoriasis? a) Absence of the stratum granulosum. b) Inflammatory cell collection. c) Thickening of the stratum spinosum. d) Retention of epidermal nuclei. Correct answer: Thickening of the stratum spinosum. What result do enlarged dermal papillae have? a) Causes supra papillary thinning. b) Increased stratum corneum thickness. c) Reduction in T-cell numbers. d) Decreased blood vessel dilation. Correct answer: Causes supra papillary thinning. What is a result of dilated and tortuous dermal vessels? a) Improved healing of lesions. b) Reduced visibility of papillae. c) Increased collagen production. d) Pinpoint bleeding when scraped. Correct answer: Pinpoint bleeding when scraped. In which layer are Munro's microabscesses generally found? a) Stratum basale. b) Stratum granulosum. c) Stratum spinosum. d) Stratum corneum. Correct answer: Stratum corneum. What is the treatment decision based on for psoriasis? a) This patient's age and gender. b) Type of psoriasis lesion only. c) Duration of psoriasis symptoms. d) Body Surface Area (BSA) involvement. Correct answer: Body Surface Area (BSA) involvement. Which treatment is recommended when BSA is less than 10%? a) Topical treatments. b) Phototherapy only. c) Systemic agents alone. d) Biological therapies exclusively. Correct answer: Topical treatments. Which drug is teratogenic and used for psoriasis vulgaris? a) Acitretin. b) Etanercept. c) Methotrexate. d) Ustekinumab. Correct answer: Methotrexate. Which biological agent targets IL-12 and IL-23? a) Secukinumab. b) Ustekinumab. c) Adalimumab. d) Guselkumab. Correct answer: Ustekinumab. Which condition is psoriasis NOT known to involve? a) Skin and mucosa. b) Central nervous system. c) Dermis layers. d) Hair follicles. Correct answer: Central nervous system. What is a distinctive negative symptom of psoriasis? a) Erosion of skin. b) No itching. c) Alopecia present. d) Auspitz sign positive. Correct answer: No itching. What defines lichen planus? a) Chronic immune-mediated inflammatory disease. b) Acute systemic infection. c) Viral skin infection. d) Autoimmune deficiency syndrome. Correct answer: Chronic immune-mediated inflammatory disease. Which factor is crucial in provoking lichen planus? a) Sunlight exposure. b) Stress and anxiety. c) Presence of mercury in dental amalgam. d) Allergy to latex. Correct answer: Presence of mercury in dental amalgam. Oral involvement in lichen planus is characterized by: a) Single ulcers with redness. b) Blisters filled with pus. c) A reticulate lacy white pattern. d) Thickened oral mucosa only. Correct answer: A reticulate lacy white pattern. What is noted about ulcerative/erosive lichen planus? a) Considered a premalignant lesion. b) Generally resolves without treatment. c) Only affects skin lesions. d) Involves no risk of malignancy. Correct answer: Considered a premalignant lesion. What characterizes psoriasis as a skin condition? a) Inflammation causing hair loss on the scalp. b) Bacterial infection resulting in pus accumulation. c) Allergic reaction to sunlight exposure. d) Rapid growth of skin cells leading to scaling. Correct answer: Rapid growth of skin cells leading to scaling. What is a defining feature of scalp psoriasis? a) Red, itchy patches with hair loss. b) Thick micaceous scales without alopecia. c) Severe dryness and flaking on the face. d) Blisters that ooze and crust over time. Correct answer: Thick micaceous scales without alopecia. How does inverse flexural psoriasis present? a) Severe redness and swelling of lesions. b) Thickened skin with significant scaling. c) Scaling is absent. d) Presence of blisters and crusting. Correct answer: Scaling is absent. What is hyperkeratosis? a) Pathological thickening of the outer skin layer. b) Loss of hair follicles in the skin. c) Retention of hair shafts within the skin. d) Fluid accumulation beneath the skin surface. Correct answer: Pathological thickening of the outer skin layer. What pathological feature is characterized by the retention of nuclei? a) Hypergranulosis associated with psoriasis. b) Absence of the granular layer in the stratum. c) Parakeratosis. d) Infiltration of lymphocytes in the dermis. Correct answer: Parakeratosis. Which condition may show absent or decreased granular layer? a) Contact dermatitis. b) Melanoma. c) Psoriasis vulgaris. d) Seborrheic dermatitis. Correct answer: Psoriasis vulgaris. What signifies the presence of Munroe's microabscess? a) Lymphocytes in the stratum basale. b) Neutrophils in the stratum corneum. c) Eosinophils in the epidermis. d) Basophils in the dermal layer. Correct answer: Neutrophils in the stratum corneum. What is associated with spongiform pustules of Kogoj? a) Eosinophils in the stratum corneum. b) Neutrophils found in the stratum spinosum. c) Basophils in the dermis. d) Macrophages in the stratum basale. Correct answer: Neutrophils found in the stratum spinosum. Which features are often seen together in psoriasis? a) Color changes and itching. b) Blistering and peeling skin. c) Absence of skin lesions. d) Parakeratosis and microabscesses. Correct answer: Parakeratosis and microabscesses. What are the small raised lesions in lichen planus called? a) Plates b) Bumps c) Warts d) Papules Correct answer: Papules What are Wickham's Striae? a) Whitish linear streaks on lesions b) Red patches on skin c) Itchy bumps on scalp d) Dark spots on nails Correct answer: Whitish linear streaks on lesions What does a positive Koebner's Phenomenon indicate? a) Lesions develop at sites of trauma b) Nail loss occurs c) Lesions are painful d) Skin heals rapidly Correct answer: Lesions develop at sites of trauma What is the most common finding of nail involvement in lichen planus? a) Thinning of the nail plate b) Ridges on the nail c) Coloration changes d) Nail plate uplift Correct answer: Thinning of the nail plate What is pterygium in lichen planus? a) A wing-shaped fold from proximal nail fold b) Thinning of hair c) Linear skin lesions d) Inflammation of nail bed Correct answer: A wing-shaped fold from proximal nail fold What type of hair loss is associated with lichen planus? a) Temporary hair loss b) Patchy hair loss c) Scarring alopecia d) Thinning hair Correct answer: Scarring alopecia What histological feature is characteristic of lichen planopilaris activity? a) Thinning epidermis b) Saw-toothing of rete ridges c) Nodular growths d) Absence of collagen Correct answer: Saw-toothing of rete ridges What is hyperkeratosis? a) Absence of hair follicles b) Shrinking of cell layers c) Thickening of the stratum corneum d) Increase in fat tissue Correct answer: Thickening of the stratum corneum Where is the band-like lymphocytic infiltrate found in lichen planus? a) Hair follicles b) Dermal-epidermal junction c) Subcutaneous layer d) Stratum corneum Correct answer: Dermal-epidermal junction What coloration do lesions typically have in lichen planus? a) Purple b) Red c) Green d) Blue Correct answer: Purple Which treatment is used for limited lichen planus? a) Antibiotics b) Surgery c) Topical steroids d) Radiation therapy Correct answer: Topical steroids What does the Pup Tent Sign indicate in lichen planus? a) Characteristic clinical sign related to lesions b) Nail healing process c) Severity of infection d) Type of inflammation Correct answer: Characteristic clinical sign related to lesions What is the histological appearance of necrotic keratinocytes? a) Presence of colloid bodies b) Epidermal atrophy c) Intact hair follicles d) Decreased vascularization Correct answer: Presence of colloid bodies What is Hyperkeratosis in the context of Lichen Planus? a) Thinning of the epidermis layer. b) Inflammation of the dermal layer. c) Increased melanin production in skin. d) Thickening of the outer layer of the skin. Correct answer: Thickening of the outer layer of the skin. What does Wedge-shaped Hypergranulosis indicate in Lichen Planus? a) Increased granule layer in the stratum granulosum. b) Decrease of cells in the epidermis. c) Thinning of the stratum corneum. d) Presence of necrotic keratinocytes. Correct answer: Increased granule layer in the stratum granulosum. Acanthosis in Lichen Planus indicates what? a) Loss of the basal cell layer. b) Inflammation of the dermis. c) Excessive scaling on the surface. d) Thickening of the stratum spinosum. Correct answer: Thickening of the stratum spinosum. What does the Sawtoothing of Rete Ridges signify? a) Absence of dermal papillae. b) Destruction of skin cells. c) Characteristic appearance of the dermal-epidermal junction. d) Improvement of the stratum corneum. Correct answer: Characteristic appearance of the dermal-epidermal junction. What does a Band-like Lymphocytic Infiltrate indicate? a) Presence of lymphocytes at the dermal-epidermal junction. b) Presence of macrophages in the dermis. c) Accumulation of red blood cells. d) Absence of inflammatory cells. Correct answer: Presence of lymphocytes at the dermal-epidermal junction. What does Basal Cell Degeneration refer to? a) Loss of the outer skin layer. b) Damage to the basal layer of the epidermis. c) Inflammation of hair follicles. d) Thickening of dermal layers. Correct answer: Damage to the basal layer of the epidermis. What are Necrotic Keratinocytes often referred to as? a) Inflammatory cells in the dermis. b) Colloid bodies or Civatte bodies. c) Living skin cells in the epidermis. d) Pigmented cells in the outer layer. Correct answer: Colloid bodies or Civatte bodies. What is an Artefactual Cleft? a) Space at the dermal-epidermal junction known as Max Joseph space. b) Layer of dead skin cells. c) Skip lesions in the dermis. d) Thickened areas of pigmentation. Correct answer: Space at the dermal-epidermal junction known as Max Joseph space. Where do lesions of Lichen Planus primarily occur? a) On extensor surfaces. b) On flexor surfaces of the body. c) In hair follicles. d) On the palms of the hands. Correct answer: On flexor surfaces of the body. What is the appearance of Lichen Planus lesions? a) Purple, pruritic, plane-topped, polygonal papules. b) Red, scaly patches on the skin. c) Round, flat, white plaques. d) Yellow, crusty, raised bumps. Correct answer: Purple, pruritic, plane-topped, polygonal papules. What is a Pterygium in Lichen Planus? a) Lack of hair growth. b) Severe discoloration of nails. c) Thickening of the cuticle. d) Skin grows over the nail bed. Correct answer: Skin grows over the nail bed. What does the Pup tent sign indicate? a) Dullness in skin color. b) Necrosis in the epidermis. c) A clinical sign observed in lesions. d) Presence of pus in blisters. Correct answer: A clinical sign observed in lesions. How do lesions of Lichen Planus heal? a) With scarring. b) Without any changes. c) With hyperpigmentation. d) With complete depigmentation. Correct answer: With hyperpigmentation. What is the first-line treatment for Limited Disease of Lichen Planus? a) Oral antibiotics. b) Chemotherapy agents. c) Mineral supplements. d) Topical Steroids. Correct answer: Topical Steroids. What treatment is used for Hypertrophic Lichen Planus? a) Systemic antibiotics. b) Topical antifungals. c) Oral antivirals. d) Intralesional Steroids (ILS). Correct answer: Intralesional Steroids (ILS). What is a treatment option for Widespread Disease of Lichen Planus? a) Systemic Steroids. b) Surgical excision. c) Cryotherapy. d) Radiation therapy. Correct answer: Systemic Steroids. What adjunct therapy may be used for Lichen Planus? a) Phototherapy. b) Active surveillance only. c) Antihistamines. d) Bacterial vaccines. Correct answer: Phototherapy. Which medication is considered a Steroid-Sparing Agent? a) Corticosteroids. b) Topical lotions. c) Biologic therapies. d) Methotrexate (Mtx). Correct answer: Methotrexate (Mtx). Which of the following is true regarding Lichen Planus and Psoriasis? a) Both are primarily infectious diseases. b) Both are classified under papulo-squamous disorders. c) Psoriasis is an acute self-limiting disease. d) Lichen Planus causes only hair loss. Correct answer: Both are classified under papulo-squamous disorders. In terms of genetic factors, which is true? a) Lichen Planus has a positive family history. b) Psoriasis is genetically more common. c) Lichen Planus is unrelated to genetics. d) No genetic factors in either disease. Correct answer: Lichen Planus has a positive family history. How does itching compare in Lichen Planus and Psoriasis? a) Psoriasis itches more than Lichen Planus. b) Lichen Planus does not cause itching. c) Both conditions never itch. d) Lichen Planus commonly itches; Psoriasis less so. Correct answer: Lichen Planus commonly itches; Psoriasis less so. What indicates oral and hair involvement in Lichen Planus? a) Hair loss with scarring alopecia occurs. b) Leukoplakia lesions only. c) Wickham's striae is present. d) No oral manifestations at all. Correct answer: Wickham's striae is present. What histopathological feature is absent in Lichen Planus? a) Parakeratosis. b) Irregular Acanthosis. c) Prominent Interface Dermatitis. d) Melanin Incontinence. Correct answer: Parakeratosis. What histological finding is specific to Psoriasis? a) Munro's Microabscess is present. b) Band-like infiltrate. c) Civatte bodies. d) Band-like degeneration. Correct answer: Munro's Microabscess is present. What is the nature of Pityriasis Rosea? a) Chronic skin condition requiring long-term treatment. b) Persistent infection lasting over a year. c) Acute self-limiting disease lasting 6-12 weeks. d) Common allergic reaction to food. Correct answer: Acute self-limiting disease lasting 6-12 weeks. What most commonly triggers Pityriasis Rosea? a) Viral infection, specifically HHV-7. b) Bacterial infections. c) Fungal infections. d) Chemical irritants. Correct answer: Viral infection, specifically HHV-7. What does the initial lesion of Pityriasis Rosea resemble? a) Flat red macules. b) Thick raised plaques. c) Linear streaks on the skin. d) Annular herald patch (mother patch). Correct answer: Annular herald patch (mother patch). When do secondary lesions develop in Pityriasis Rosea? a) 1-2 weeks after the initial lesion. b) Immediately after the first patch. c) Several months later. d) Only when infected. Correct answer: 1-2 weeks after the initial lesion. What characterizes pigmentary changes in Pityriasis Rosea? a) Dark blue spots that persist indefinitely. b) Mild changes that fade slowly over time. c) Severe discoloration requiring treatment. d) Scarring leading to skin texture changes. Correct answer: Mild changes that fade slowly over time. What is a key characteristic of Lichen Planus? a) Lesions are always painless. b) Lesions appear only red. c) Lesions are often itchy. d) Lesions are round in shape. Correct answer: Lesions are often itchy. What color do Lichen Planus lesions typically appear? a) Purple b) Green c) Yellow d) Brown Correct answer: Purple How are lesions of Lichen Planus described in shape? a) Polygonal b) Spherical c) Irregular d) Elliptical Correct answer: Polygonal What is the appearance of Lichen Planus lesions? a) Bumpy b) Indentation c) Rough-edged d) Plain topped Correct answer: Plain topped What type of lesion in Lichen Planus is described as small, raised bumps? a) Plaques b) Nodules c) Cysts d) Papules Correct answer: Papules What do Wickham's striae appear as on Lichen Planus lesions? a) Whitish linear streaks b) Red spots c) Yellow patches d) Black lines Correct answer: Whitish linear streaks What does a positive Koebner's phenomenon indicate in Lichen Planus? a) Skin is unaffected by trauma. b) New lesions develop at trauma sites. c) No new lesions form. d) Only old lesions are maintained. Correct answer: New lesions develop at trauma sites. What is the most frequent observation of nail involvement in Lichen Planus? a) Complete nail loss b) Thinning of the nail plate c) Nail discoloration d) Nail thickening Correct answer: Thinning of the nail plate What is pterygium in relation to Lichen Planus? a) Excessive nail growth b) A wing-shaped fold extending from the nail fold. c) A nail bed infection d) A type of nail polish Correct answer: A wing-shaped fold extending from the nail fold. What should patients be aware of regarding hair involvement in Lichen Planus? a) Increased hair growth b) They may experience hair loss. c) No change in hair d) Only scalp hair is affected Correct answer: They may experience hair loss. What histological feature is characteristic of Lichen Planopilaris? a) Smooth epidermis b) Flat ridges c) Deep grooves d) Saw-toothing rete ridges Correct answer: Saw-toothing rete ridges What describes perifollicular lichenoid papules in Lichen Planopilaris? a) Acne around follicles b) Cysts on the scalp c) Small, raised lesions around hair follicles. d) Large bumps on the scalp Correct answer: Small, raised lesions around hair follicles. What does the term 'Collarette' refer to in cerebellum pathology? a) A collar-like appearance of lesions. b) A type of skin texture description. c) A scale's color gradient indication. d) A pattern of hair loss. Correct answer: A collar-like appearance of lesions. Which pattern do lesions typically follow in the cerebellum? a) Spiral pattern. b) Christmas tree pattern. c) Grid pattern. d) Dendritic pattern. Correct answer: Christmas tree pattern. How do lesions spread from the spine in cerebellar conditions? a) Upwards and inwards towards the spine. b) Downwards and outwards following lines of Langer. c) Radially outward from the center. d) Only upwards following skin folds. Correct answer: Downwards and outwards following lines of Langer. What characterizes hyperkeratotic perifollicular papules in PRP? a) Smooth velvet texture. b) Nutmeg Grater appearance. c) Crusty scab formation. d) Shiny flat lesions. Correct answer: Nutmeg Grater appearance. What is the progression of lesions in Pityriasis Rubra Pilaris? a) From discrete to confluent orange-red erythema. b) From confluent to smaller discrete spots. c) From red to black lesions. d) Random spread across the body. Correct answer: From discrete to confluent orange-red erythema. Which area of the body is notably affected in PRP? a) Face and neck with acne. b) Palms and soles leading to palmoplantar keratoderma. c) Scalp leading to dandruff. d) Back leading to scarring. Correct answer: Palms and soles leading to palmoplantar keratoderma. What is eczema primarily a response to? a) Only bacterial infections. b) Endogenous or exogenous stimuli. c) Seasonal allergies. d) Genetic skin conditions. Correct answer: Endogenous or exogenous stimuli. Which condition is classified under exogenous eczema? a) Seborrheic dermatitis. b) Stasis eczema. c) Contact dermatitis. d) Nummular eczema. Correct answer: Contact dermatitis. Which type of dermatitis affects oily areas of the body? a) Asteatotic eczema. b) Seborrheic dermatitis. c) Pompholyx. d) Pityriasis alba. Correct answer: Seborrheic dermatitis. What is a key characteristic of nummular eczema? a) Patchy dryness across the body. b) Coin-shaped spots of eczema. c) Blisters on the hands. d) Hyperpigmented areas on the face. Correct answer: Coin-shaped spots of eczema. What is a defining feature of pompholyx? a) Rough patches on elbows. b) Red rash on the torso. c) Blistering eczema on hands and feet. d) Scaly skin on the scalp. Correct answer: Blistering eczema on hands and feet. What characterizes acute eczema? a) Erythema, edema, vesicles, oozing, and crusting. b) Persistent scaling and thickening of skin. c) Chronic itching and darkening of skin. d) Only mild redness with no swelling. Correct answer: Erythema, edema, vesicles, oozing, and crusting. What is a defining feature of chronic eczema? a) Small fluid-filled blisters. b) Formation of crust over lesions. c) Chronic itching with thickened skin. d) Persistent redness and scaling. Correct answer: Chronic itching with thickened skin. Which type of dermatitis is non-immunologic? a) Allergic Contact Dermatitis (ACD). b) Chronic Eczema. c) Acute Eczema. d) Irritant Contact Dermatitis (ICD). Correct answer: Irritant Contact Dermatitis (ICD). What is spongiosis? a) Thickening of skin with markings. b) Formation of small fluid-filled blisters. c) Intercellular edema between skin cells. d) Direct tissue damage from irritants. Correct answer: Intercellular edema between skin cells. What does subacute eczema primarily exhibit? a) Chronic itching and hyperpigmentation. b) Fluid discharge and crusting. c) Only thickened skin with no redness. d) Persistent erythema and scaling. Correct answer: Persistent erythema and scaling. Who is affected by allergic contact dermatitis (ACD)? a) All individuals exposed to irritants. b) Only infants and children. c) Genetically predisposed individuals (Atopy positive). d) Individuals allergic to sun exposure. Correct answer: Genetically predisposed individuals (Atopy positive). Lesions from ICD are typically: a) Restricted to the site of contact. b) Disseminated beyond the contact site. c) Characterized by blistering rash. d) Always present without any irritant exposure. Correct answer: Restricted to the site of contact. What type of sensation is more common in allergic contact dermatitis? a) More burning sensation. b) No sensation at all. c) More itching sensation. d) Dull pain or discomfort. Correct answer: More itching sensation. Diaper dermatitis primarily affects which areas? a) Convex areas while sparing skin folds. b) All skin folds and creases. c) Only the abdominal region. d) The entire body evenly. Correct answer: Convex areas while sparing skin folds. What chemical is known to cause hair dye reactions? a) Para-phenylenediamine (PPD). b) Benzoyl peroxide. c) Hydrochloric acid. d) Sodium chloride. Correct answer: Para-phenylenediamine (PPD). What diagnostic method is used to identify allergens causing ACD? a) Blood test. b) Skin biopsy. c) Urine analysis. d) Patch Test. Correct answer: Patch Test. What is the primary allergen in hair dye that causes ACD? a) Dimethicone b) Cetearyl alcohol c) PPD (para phenylene diamine) d) Ethylhexyl methoxycinnamate Correct answer: PPD (para phenylene diamine) Which metal is commonly associated with allergic contact dermatitis? a) Iron b) Aluminum c) Copper d) Nickel Correct answer: Nickel What is the allergen found in topical antibiotics linked to ACD? a) Bacitracin b) Neomycin c) Clindamycin d) Tetracycline Correct answer: Neomycin What allergen in cement can cause allergic contact dermatitis? a) Sodium sulfamate b) Potassium dichromate c) Calcium carbonate d) Boric acid Correct answer: Potassium dichromate What is the key allergen in a bindi? a) PTBP (para tertiary butyl phenol) b) Salicylic acid c) Benzyl alcohol d) Linalool Correct answer: PTBP (para tertiary butyl phenol) What type of hypersensitivity reaction is involved in a patch test? a) Type 4 Hypersensitivity Reaction b) Type 1 Hypersensitivity Reaction c) Type 2 Hypersensitivity Reaction d) Type 3 Hypersensitivity Reaction Correct answer: Type 4 Hypersensitivity Reaction When is the best reading time for a patch test? a) 24 hours (1 day) b) 72 hours (3 days) c) 96 hours (4 days) d) 120 hours (5 days) Correct answer: 96 hours (4 days) What allergen is associated with airborne contact dermatitis? a) Histamine b) Formaldehyde c) Sesquiterpene lactone (SQL) d) Acetaldehyde Correct answer: Sesquiterpene lactone (SQL) What plant is a common source of sesquiterpene lactone? a) Parthenium hysterophorus b) Aloe vera c) Eucalyptus globulus d) Ivy gourd Correct answer: Parthenium hysterophorus What is the primary cause of seborrheic dermatitis? a) Bacteria b) Viruses c) Malassezia yeast d) Dust mites Correct answer: Malassezia yeast Where is seborrheic dermatitis typically found? a) Armpits b) Hands c) Scalp d) Back Correct answer: Scalp What is a characteristic of scales seen in seborrheic dermatitis? a) White dry flakes b) Greasy yellow scales c) Red raised bumps d) Brown crusty patches Correct answer: Greasy yellow scales Which disease states are associated with severe seborrheic dermatitis? a) Diabetes and Hypertension b) Asthma and Allergies c) Parkinson's disease and HIV infection d) Obesity and Arthritis Correct answer: Parkinson's disease and HIV infection What is the common name for infantile seborrheic dermatitis? a) Baby acne b) Diaper rash c) Atopic eczema d) Cradle Cap Correct answer: Cradle Cap What is stasis eczema also known as? a) Contact dermatitis b) Inflammatory dermatitis c) Gravitational or varicose eczema d) Allergic dermatitis Correct answer: Gravitational or varicose eczema Stasis eczema is linked to what condition? a) Asthma b) Chronic venous insufficiency c) Peripheral artery disease d) Diabetes mellitus Correct answer: Chronic venous insufficiency What is a characteristic appearance of stasis eczema? a) Psoriatic plaques b) Lichenification c) Erythema multiforme d) Lipodermatosclerosis Correct answer: Lipodermatosclerosis How does lipodermatosclerosis appear? a) Inverted champagne bottle appearance b) Pear-shaped appearance c) Round disc-like appearance d) Cylindrical appearance Correct answer: Inverted champagne bottle appearance What characterizes Pityriasis Alba? a) Coin-shaped lesions with well-demarcated edges. b) Hypopigmented patches, commonly seen in children. c) Intensely pruritic deep-seated vesicles. d) Lichenified flexural lesions prevalent in adults. Correct answer: Hypopigmented patches, commonly seen in children. What is a key characteristic of Nummular Eczema? a) Hypopigmented patches on the cheeks. b) Coin-shaped lesions with well-demarcated edges. c) Intensely painful vesicles on digits. d) Exudative lesions that are bilaterally symmetrical. Correct answer: Coin-shaped lesions with well-demarcated edges. Which feature distinguishes Pompholyx? a) Coin-shaped lesions with clear edges. b) Intensely pruritic and painful deep-seated vesicles. c) Exudative, symmetrical lesions on the face. d) Lichenified lesions found in flexural areas. Correct answer: Intensely pruritic and painful deep-seated vesicles. Atopic Dermatitis is classified as what type of skin disease? a) A contagious skin infection. b) An endogenous eczema. c) A form of dermatitis limited to adults. d) An acute allergic reaction to allergens. Correct answer: An endogenous eczema. What does the Atopic Triad consist of? a) Eczema, Psoriasis, Acne. b) Recurrent Allergic Rhinitis, Atopic Asthma, Skin Allergies. c) Asthma, Hay Fever, Cold Sores. d) Dry Skin, Dermatitis, Rosacea. Correct answer: Recurrent Allergic Rhinitis, Atopic Asthma, Skin Allergies. What are the clinical features of the Infantile Phase of Atopic Dermatitis? a) Lichenified lesions mainly on the hands. b) Coin-shaped lesions on the trunk. c) Extremely itchy, exudative lesions, bilaterally symmetrical. d) Severe red papules on the feet. Correct answer: Extremely itchy, exudative lesions, bilaterally symmetrical. In what area do Childhood Phase Atopic Dermatitis lesions typically appear? a) Cubital and popliteal fossa (flexures). b) Areas behind the ears. c) The diaper area of infants. d) On the soles of the feet. Correct answer: Cubital and popliteal fossa (flexures). How do Adult Phase Atopic Dermatitis lesions differ from earlier phases? a) Lichenified flexural lesions, less common. b) More frequent exudative lesions. c) Highly symmetrical red plaques. d) Relatively more superficial lesions. Correct answer: Lichenified flexural lesions, less common. What is the prognosis for Infantile Phase Atopic Dermatitis by 18 months? a) Clears in 40% of cases. b) Worsens in 60% of cases. c) Remains active in all cases. d) Clears in 80% of cases. Correct answer: Clears in 40% of cases. What is the sensation like in Pityriasis Alba? a) Sensation is typically intact. b) Sensation is hypersensitive. c) Sensation may be diminished. d) Sensation is usually painful. Correct answer: Sensation is typically intact. What must be differentiated from Nummular Eczema? a) Psoriasis. b) Pityriasis Alba. c) Tinea. d) Rosacea. Correct answer: Tinea. What phase engages the extensor muscles in infants? a) Walking phase of toddlers. b) Sitting phase of infants. c) Crawling phase of toddlers. d) Crawling phase of infants. Correct answer: Crawling phase of infants. What is the necessary investigation for diagnosing skin conditions? a) Blood tests and imaging scans. b) Patient history and questionnaires. c) Skin biopsies and allergen tests. d) Thorough clinical examination. Correct answer: Thorough clinical examination. What is the hallmark symptom of atopic dermatitis? a) Persistent redness of the skin. b) Dry skin without itching. c) Intermittent and intense pruritus. d) Rash that appears only during summer. Correct answer: Intermittent and intense pruritus. How does atopic dermatitis present in individuals? a) Only as chronic lesions. b) With only one type of rash. c) Through occasional blisters. d) In three distinct phases. Correct answer: In three distinct phases. What indicates a chronic or chronically relapsing dermatitis? a) The rash disappears entirely. b) Only appears after stress. c) Changes color frequently. d) Condition may persist or recur over time. Correct answer: Condition may persist or recur over time. What is significant for diagnosing atopic dermatitis? a) History of vaccinations. b) Recent travel history. c) Previous surgical interventions. d) Personal or family history of atopy. Correct answer: Personal or family history of atopy. What does xerosis refer to? a) Dry skin. b) Oily skin. c) Red and inflamed skin. d) Scaly skin with blisters. Correct answer: Dry skin. What is ichthyosis characterized by? a) Thickened patches of skin. b) Bumps and rashes on the skin. c) Fish-like scales on the skin. d) Flaky skin shedding in sheets. Correct answer: Fish-like scales on the skin. What does white dermographism indicate? a) A white line appears on stroking skin. b) A red flush on the skin. c) A swelling appears on stroke. d) Dark pigmentation on the skin. Correct answer: A white line appears on stroking skin. What does the headlight sign indicate? a) Swelling around the eyes. b) Sparing of the periorbital areas. c) Rash on the forehead. d) Redness of the conjunctiva. Correct answer: Sparing of the periorbital areas. What does keratoconus affect? a) The color of the iris. b) The clarity of the lens. c) The shape of the cornea. d) The pupil's diameter. Correct answer: The shape of the cornea. What are allergic shiners? a) Dark circles under the eyes due to allergies. b) Swelling of the eyelids. c) Redness of the eyes. d) Itching around the eyes. Correct answer: Dark circles under the eyes due to allergies. What is the allergic salute? a) Sneezing and coughing. b) Rubbing the nose due to allergies. c) Touching the face frequently. d) Swelling of the nose

Use Quizgecko on...
Browser
Browser